Finding a particular solution to a linear PDE












7












$begingroup$


I want to solve the PDE
$$frac{partial u}{partial t}+x_1(x_2-x_3) frac{partial u}{partial x_1}+x_2(x_3-x_1) frac{partial u}{partial x_2}+x_3(x_1-x_2) frac{partial u}{partial x_3}=sum_{i=1}^3 alpha_i frac{partial f}{partial x_i}, tag{1} $$
where $alpha_1,alpha_2,alpha_3$ are constants and $f$ is the function



$$f(mathbf{x},t)= frac{alpha _1 left(wp 'left(t;g_2,g_3right)+x_2x_3 left(x_2-x_3right)right)}{2 left(wp left(t;g_2,g_3right)-frac{1}{12} left(x_1+x_2+x_3right){}^2+x_2 x_3right)}+frac{alpha_2 left(wp 'left(t;g_2,g_3right)+x_1 x_3 left(x_3-x_1right)right)}{2 left(wp left(t;g_2,g_3right)-frac{1}{12} left(x_1+x_2+x_3right){}^2+x_1 x_3right)}+frac{alpha _3 left(wp 'left(t;g_2,g_3right)+x_1x_2 left(x_1-x_2right) right)}{2 left(wp left(t;g_2,g_3right)-frac{1}{12} left(x_1+x_2+x_3right){}^2+x_1 x_2right)}+left(alpha _1+alpha _2+alpha _3right) left(zeta left(t;g_2,g_3right)+frac{1}{12} t left(x_1+x_2+x_3right){}^2right). $$



Here $wp$ and $zeta$ are the Weierstraß p- and zeta- functions respectively, with the elliptic invariants
$$ begin{align}
g_2 &= frac{(x_1+x_2+x_3)^4}{12}-2 x_1 x_2 x_3 (x_1+x_2+x_3), \
g_3 &= -(x_1 x_2 x_3)^2+frac{x_1 x_2 x_3 (x_1+x_2+x_3)^3}{6}-frac{(x_1+x_2+x_3)^6}{216}.
end{align} $$



From this point onward the invariants $g_2,g_3$ will not be shown explicitly.





My attempt:
First, I managed to solve the associated homogeneous PDE
$$ frac{partial u_h}{partial t}+x_1(x_2-x_3) frac{partial u_h}{partial x_1}+x_2(x_3-x_1) frac{partial u_h}{partial x_2}+x_3(x_1-x_2) frac{partial u_h}{partial x_3}=0, $$ via the method of characteristics. The solution is given by
$$ u_h =Phi left( X_1(mathbf{x},t), X_2(mathbf{x},t) ,X_3(mathbf{x},t) right) $$
where $Phi$ is an arbitrary function, and
$$X_1=frac{12 x_1 x_2 x_3}{left(x_1+x_2+x_3right)^2-12 left(frac{left(wp'(t)-x_{2}x_{3} left(x_{3}-x_{2}right) right)^2}{4 left(wp (t)+ x_{2}x_{3} -frac{1}{12} left(x_1+x_2+x_3right)^2right)^2}-wp (t)+ x_{2} x_{3}-frac{1}{12} left(x_1+x_2+x_3right)^2right)}, \
X_2 = frac{12 x_1 x_2 x_3}{left(x_1+x_2+x_3right)^2-12 left(frac{left(wp'(t)-x_{1} x_{3} left(x_{1}-x_{3}right) right)^2}{4 left(wp (t)+x_{1} x_{3}-frac{1}{12} left(x_1+x_2+x_3right)^2right)^2}-wp (t)+x_{1} x_{3}-frac{1}{12} left(x_1+x_2+x_3right)^2right)}, \
X_3 = frac{12 x_1 x_2 x_3}{left(x_1+x_2+x_3right)^2-12 left(frac{left(wp'(t)-x_{1} x_{2} left(x_{2}-x_{1}right) right)^2}{4 left(wp (t)+ x_{1} x_{2}-frac{1}{12} left(x_1+x_2+x_3right)^2right)^2}-wp (t)+ x_{1} x_{2}-frac{1}{12} left(x_1+x_2+x_3right)^2right)}. $$



The final ingredient is a particular solution. Denoting the RHS in Equation (1) above by $R(mathbf{x},t)$, Duhamel's principle (or the method of characteristics again) suggests that a particular solution is given by
$$u_p=int_0^t R left( mathbf{X}(mathbf{x},t-u),u right) mathrm{d} u .$$
I tried computing this with Mathematica and it didn't go well. This is probably because Mathematica seems to be unaware of the elliptic identity $wp'^2=4 wp^3-g_2 wp -g_3$.



I would appreciate help with the evaluation of the integral above, or any other method of obtaining a particular solution of Equation (1).



Thank you!










share|cite|improve this question









$endgroup$












  • $begingroup$
    You already know that a solution exists. This is not enough?
    $endgroup$
    – timur
    Dec 26 '18 at 0:38










  • $begingroup$
    @timur No, I want to see if I can get it explicitly.
    $endgroup$
    – user1337
    Dec 26 '18 at 9:07
















7












$begingroup$


I want to solve the PDE
$$frac{partial u}{partial t}+x_1(x_2-x_3) frac{partial u}{partial x_1}+x_2(x_3-x_1) frac{partial u}{partial x_2}+x_3(x_1-x_2) frac{partial u}{partial x_3}=sum_{i=1}^3 alpha_i frac{partial f}{partial x_i}, tag{1} $$
where $alpha_1,alpha_2,alpha_3$ are constants and $f$ is the function



$$f(mathbf{x},t)= frac{alpha _1 left(wp 'left(t;g_2,g_3right)+x_2x_3 left(x_2-x_3right)right)}{2 left(wp left(t;g_2,g_3right)-frac{1}{12} left(x_1+x_2+x_3right){}^2+x_2 x_3right)}+frac{alpha_2 left(wp 'left(t;g_2,g_3right)+x_1 x_3 left(x_3-x_1right)right)}{2 left(wp left(t;g_2,g_3right)-frac{1}{12} left(x_1+x_2+x_3right){}^2+x_1 x_3right)}+frac{alpha _3 left(wp 'left(t;g_2,g_3right)+x_1x_2 left(x_1-x_2right) right)}{2 left(wp left(t;g_2,g_3right)-frac{1}{12} left(x_1+x_2+x_3right){}^2+x_1 x_2right)}+left(alpha _1+alpha _2+alpha _3right) left(zeta left(t;g_2,g_3right)+frac{1}{12} t left(x_1+x_2+x_3right){}^2right). $$



Here $wp$ and $zeta$ are the Weierstraß p- and zeta- functions respectively, with the elliptic invariants
$$ begin{align}
g_2 &= frac{(x_1+x_2+x_3)^4}{12}-2 x_1 x_2 x_3 (x_1+x_2+x_3), \
g_3 &= -(x_1 x_2 x_3)^2+frac{x_1 x_2 x_3 (x_1+x_2+x_3)^3}{6}-frac{(x_1+x_2+x_3)^6}{216}.
end{align} $$



From this point onward the invariants $g_2,g_3$ will not be shown explicitly.





My attempt:
First, I managed to solve the associated homogeneous PDE
$$ frac{partial u_h}{partial t}+x_1(x_2-x_3) frac{partial u_h}{partial x_1}+x_2(x_3-x_1) frac{partial u_h}{partial x_2}+x_3(x_1-x_2) frac{partial u_h}{partial x_3}=0, $$ via the method of characteristics. The solution is given by
$$ u_h =Phi left( X_1(mathbf{x},t), X_2(mathbf{x},t) ,X_3(mathbf{x},t) right) $$
where $Phi$ is an arbitrary function, and
$$X_1=frac{12 x_1 x_2 x_3}{left(x_1+x_2+x_3right)^2-12 left(frac{left(wp'(t)-x_{2}x_{3} left(x_{3}-x_{2}right) right)^2}{4 left(wp (t)+ x_{2}x_{3} -frac{1}{12} left(x_1+x_2+x_3right)^2right)^2}-wp (t)+ x_{2} x_{3}-frac{1}{12} left(x_1+x_2+x_3right)^2right)}, \
X_2 = frac{12 x_1 x_2 x_3}{left(x_1+x_2+x_3right)^2-12 left(frac{left(wp'(t)-x_{1} x_{3} left(x_{1}-x_{3}right) right)^2}{4 left(wp (t)+x_{1} x_{3}-frac{1}{12} left(x_1+x_2+x_3right)^2right)^2}-wp (t)+x_{1} x_{3}-frac{1}{12} left(x_1+x_2+x_3right)^2right)}, \
X_3 = frac{12 x_1 x_2 x_3}{left(x_1+x_2+x_3right)^2-12 left(frac{left(wp'(t)-x_{1} x_{2} left(x_{2}-x_{1}right) right)^2}{4 left(wp (t)+ x_{1} x_{2}-frac{1}{12} left(x_1+x_2+x_3right)^2right)^2}-wp (t)+ x_{1} x_{2}-frac{1}{12} left(x_1+x_2+x_3right)^2right)}. $$



The final ingredient is a particular solution. Denoting the RHS in Equation (1) above by $R(mathbf{x},t)$, Duhamel's principle (or the method of characteristics again) suggests that a particular solution is given by
$$u_p=int_0^t R left( mathbf{X}(mathbf{x},t-u),u right) mathrm{d} u .$$
I tried computing this with Mathematica and it didn't go well. This is probably because Mathematica seems to be unaware of the elliptic identity $wp'^2=4 wp^3-g_2 wp -g_3$.



I would appreciate help with the evaluation of the integral above, or any other method of obtaining a particular solution of Equation (1).



Thank you!










share|cite|improve this question









$endgroup$












  • $begingroup$
    You already know that a solution exists. This is not enough?
    $endgroup$
    – timur
    Dec 26 '18 at 0:38










  • $begingroup$
    @timur No, I want to see if I can get it explicitly.
    $endgroup$
    – user1337
    Dec 26 '18 at 9:07














7












7








7


2



$begingroup$


I want to solve the PDE
$$frac{partial u}{partial t}+x_1(x_2-x_3) frac{partial u}{partial x_1}+x_2(x_3-x_1) frac{partial u}{partial x_2}+x_3(x_1-x_2) frac{partial u}{partial x_3}=sum_{i=1}^3 alpha_i frac{partial f}{partial x_i}, tag{1} $$
where $alpha_1,alpha_2,alpha_3$ are constants and $f$ is the function



$$f(mathbf{x},t)= frac{alpha _1 left(wp 'left(t;g_2,g_3right)+x_2x_3 left(x_2-x_3right)right)}{2 left(wp left(t;g_2,g_3right)-frac{1}{12} left(x_1+x_2+x_3right){}^2+x_2 x_3right)}+frac{alpha_2 left(wp 'left(t;g_2,g_3right)+x_1 x_3 left(x_3-x_1right)right)}{2 left(wp left(t;g_2,g_3right)-frac{1}{12} left(x_1+x_2+x_3right){}^2+x_1 x_3right)}+frac{alpha _3 left(wp 'left(t;g_2,g_3right)+x_1x_2 left(x_1-x_2right) right)}{2 left(wp left(t;g_2,g_3right)-frac{1}{12} left(x_1+x_2+x_3right){}^2+x_1 x_2right)}+left(alpha _1+alpha _2+alpha _3right) left(zeta left(t;g_2,g_3right)+frac{1}{12} t left(x_1+x_2+x_3right){}^2right). $$



Here $wp$ and $zeta$ are the Weierstraß p- and zeta- functions respectively, with the elliptic invariants
$$ begin{align}
g_2 &= frac{(x_1+x_2+x_3)^4}{12}-2 x_1 x_2 x_3 (x_1+x_2+x_3), \
g_3 &= -(x_1 x_2 x_3)^2+frac{x_1 x_2 x_3 (x_1+x_2+x_3)^3}{6}-frac{(x_1+x_2+x_3)^6}{216}.
end{align} $$



From this point onward the invariants $g_2,g_3$ will not be shown explicitly.





My attempt:
First, I managed to solve the associated homogeneous PDE
$$ frac{partial u_h}{partial t}+x_1(x_2-x_3) frac{partial u_h}{partial x_1}+x_2(x_3-x_1) frac{partial u_h}{partial x_2}+x_3(x_1-x_2) frac{partial u_h}{partial x_3}=0, $$ via the method of characteristics. The solution is given by
$$ u_h =Phi left( X_1(mathbf{x},t), X_2(mathbf{x},t) ,X_3(mathbf{x},t) right) $$
where $Phi$ is an arbitrary function, and
$$X_1=frac{12 x_1 x_2 x_3}{left(x_1+x_2+x_3right)^2-12 left(frac{left(wp'(t)-x_{2}x_{3} left(x_{3}-x_{2}right) right)^2}{4 left(wp (t)+ x_{2}x_{3} -frac{1}{12} left(x_1+x_2+x_3right)^2right)^2}-wp (t)+ x_{2} x_{3}-frac{1}{12} left(x_1+x_2+x_3right)^2right)}, \
X_2 = frac{12 x_1 x_2 x_3}{left(x_1+x_2+x_3right)^2-12 left(frac{left(wp'(t)-x_{1} x_{3} left(x_{1}-x_{3}right) right)^2}{4 left(wp (t)+x_{1} x_{3}-frac{1}{12} left(x_1+x_2+x_3right)^2right)^2}-wp (t)+x_{1} x_{3}-frac{1}{12} left(x_1+x_2+x_3right)^2right)}, \
X_3 = frac{12 x_1 x_2 x_3}{left(x_1+x_2+x_3right)^2-12 left(frac{left(wp'(t)-x_{1} x_{2} left(x_{2}-x_{1}right) right)^2}{4 left(wp (t)+ x_{1} x_{2}-frac{1}{12} left(x_1+x_2+x_3right)^2right)^2}-wp (t)+ x_{1} x_{2}-frac{1}{12} left(x_1+x_2+x_3right)^2right)}. $$



The final ingredient is a particular solution. Denoting the RHS in Equation (1) above by $R(mathbf{x},t)$, Duhamel's principle (or the method of characteristics again) suggests that a particular solution is given by
$$u_p=int_0^t R left( mathbf{X}(mathbf{x},t-u),u right) mathrm{d} u .$$
I tried computing this with Mathematica and it didn't go well. This is probably because Mathematica seems to be unaware of the elliptic identity $wp'^2=4 wp^3-g_2 wp -g_3$.



I would appreciate help with the evaluation of the integral above, or any other method of obtaining a particular solution of Equation (1).



Thank you!










share|cite|improve this question









$endgroup$




I want to solve the PDE
$$frac{partial u}{partial t}+x_1(x_2-x_3) frac{partial u}{partial x_1}+x_2(x_3-x_1) frac{partial u}{partial x_2}+x_3(x_1-x_2) frac{partial u}{partial x_3}=sum_{i=1}^3 alpha_i frac{partial f}{partial x_i}, tag{1} $$
where $alpha_1,alpha_2,alpha_3$ are constants and $f$ is the function



$$f(mathbf{x},t)= frac{alpha _1 left(wp 'left(t;g_2,g_3right)+x_2x_3 left(x_2-x_3right)right)}{2 left(wp left(t;g_2,g_3right)-frac{1}{12} left(x_1+x_2+x_3right){}^2+x_2 x_3right)}+frac{alpha_2 left(wp 'left(t;g_2,g_3right)+x_1 x_3 left(x_3-x_1right)right)}{2 left(wp left(t;g_2,g_3right)-frac{1}{12} left(x_1+x_2+x_3right){}^2+x_1 x_3right)}+frac{alpha _3 left(wp 'left(t;g_2,g_3right)+x_1x_2 left(x_1-x_2right) right)}{2 left(wp left(t;g_2,g_3right)-frac{1}{12} left(x_1+x_2+x_3right){}^2+x_1 x_2right)}+left(alpha _1+alpha _2+alpha _3right) left(zeta left(t;g_2,g_3right)+frac{1}{12} t left(x_1+x_2+x_3right){}^2right). $$



Here $wp$ and $zeta$ are the Weierstraß p- and zeta- functions respectively, with the elliptic invariants
$$ begin{align}
g_2 &= frac{(x_1+x_2+x_3)^4}{12}-2 x_1 x_2 x_3 (x_1+x_2+x_3), \
g_3 &= -(x_1 x_2 x_3)^2+frac{x_1 x_2 x_3 (x_1+x_2+x_3)^3}{6}-frac{(x_1+x_2+x_3)^6}{216}.
end{align} $$



From this point onward the invariants $g_2,g_3$ will not be shown explicitly.





My attempt:
First, I managed to solve the associated homogeneous PDE
$$ frac{partial u_h}{partial t}+x_1(x_2-x_3) frac{partial u_h}{partial x_1}+x_2(x_3-x_1) frac{partial u_h}{partial x_2}+x_3(x_1-x_2) frac{partial u_h}{partial x_3}=0, $$ via the method of characteristics. The solution is given by
$$ u_h =Phi left( X_1(mathbf{x},t), X_2(mathbf{x},t) ,X_3(mathbf{x},t) right) $$
where $Phi$ is an arbitrary function, and
$$X_1=frac{12 x_1 x_2 x_3}{left(x_1+x_2+x_3right)^2-12 left(frac{left(wp'(t)-x_{2}x_{3} left(x_{3}-x_{2}right) right)^2}{4 left(wp (t)+ x_{2}x_{3} -frac{1}{12} left(x_1+x_2+x_3right)^2right)^2}-wp (t)+ x_{2} x_{3}-frac{1}{12} left(x_1+x_2+x_3right)^2right)}, \
X_2 = frac{12 x_1 x_2 x_3}{left(x_1+x_2+x_3right)^2-12 left(frac{left(wp'(t)-x_{1} x_{3} left(x_{1}-x_{3}right) right)^2}{4 left(wp (t)+x_{1} x_{3}-frac{1}{12} left(x_1+x_2+x_3right)^2right)^2}-wp (t)+x_{1} x_{3}-frac{1}{12} left(x_1+x_2+x_3right)^2right)}, \
X_3 = frac{12 x_1 x_2 x_3}{left(x_1+x_2+x_3right)^2-12 left(frac{left(wp'(t)-x_{1} x_{2} left(x_{2}-x_{1}right) right)^2}{4 left(wp (t)+ x_{1} x_{2}-frac{1}{12} left(x_1+x_2+x_3right)^2right)^2}-wp (t)+ x_{1} x_{2}-frac{1}{12} left(x_1+x_2+x_3right)^2right)}. $$



The final ingredient is a particular solution. Denoting the RHS in Equation (1) above by $R(mathbf{x},t)$, Duhamel's principle (or the method of characteristics again) suggests that a particular solution is given by
$$u_p=int_0^t R left( mathbf{X}(mathbf{x},t-u),u right) mathrm{d} u .$$
I tried computing this with Mathematica and it didn't go well. This is probably because Mathematica seems to be unaware of the elliptic identity $wp'^2=4 wp^3-g_2 wp -g_3$.



I would appreciate help with the evaluation of the integral above, or any other method of obtaining a particular solution of Equation (1).



Thank you!







integration ordinary-differential-equations pde closed-form elliptic-functions






share|cite|improve this question













share|cite|improve this question











share|cite|improve this question




share|cite|improve this question










asked Dec 17 '18 at 16:21









user1337user1337

16.8k43592




16.8k43592












  • $begingroup$
    You already know that a solution exists. This is not enough?
    $endgroup$
    – timur
    Dec 26 '18 at 0:38










  • $begingroup$
    @timur No, I want to see if I can get it explicitly.
    $endgroup$
    – user1337
    Dec 26 '18 at 9:07


















  • $begingroup$
    You already know that a solution exists. This is not enough?
    $endgroup$
    – timur
    Dec 26 '18 at 0:38










  • $begingroup$
    @timur No, I want to see if I can get it explicitly.
    $endgroup$
    – user1337
    Dec 26 '18 at 9:07
















$begingroup$
You already know that a solution exists. This is not enough?
$endgroup$
– timur
Dec 26 '18 at 0:38




$begingroup$
You already know that a solution exists. This is not enough?
$endgroup$
– timur
Dec 26 '18 at 0:38












$begingroup$
@timur No, I want to see if I can get it explicitly.
$endgroup$
– user1337
Dec 26 '18 at 9:07




$begingroup$
@timur No, I want to see if I can get it explicitly.
$endgroup$
– user1337
Dec 26 '18 at 9:07










0






active

oldest

votes











Your Answer





StackExchange.ifUsing("editor", function () {
return StackExchange.using("mathjaxEditing", function () {
StackExchange.MarkdownEditor.creationCallbacks.add(function (editor, postfix) {
StackExchange.mathjaxEditing.prepareWmdForMathJax(editor, postfix, [["$", "$"], ["\\(","\\)"]]);
});
});
}, "mathjax-editing");

StackExchange.ready(function() {
var channelOptions = {
tags: "".split(" "),
id: "69"
};
initTagRenderer("".split(" "), "".split(" "), channelOptions);

StackExchange.using("externalEditor", function() {
// Have to fire editor after snippets, if snippets enabled
if (StackExchange.settings.snippets.snippetsEnabled) {
StackExchange.using("snippets", function() {
createEditor();
});
}
else {
createEditor();
}
});

function createEditor() {
StackExchange.prepareEditor({
heartbeatType: 'answer',
autoActivateHeartbeat: false,
convertImagesToLinks: true,
noModals: true,
showLowRepImageUploadWarning: true,
reputationToPostImages: 10,
bindNavPrevention: true,
postfix: "",
imageUploader: {
brandingHtml: "Powered by u003ca class="icon-imgur-white" href="https://imgur.com/"u003eu003c/au003e",
contentPolicyHtml: "User contributions licensed under u003ca href="https://creativecommons.org/licenses/by-sa/3.0/"u003ecc by-sa 3.0 with attribution requiredu003c/au003e u003ca href="https://stackoverflow.com/legal/content-policy"u003e(content policy)u003c/au003e",
allowUrls: true
},
noCode: true, onDemand: true,
discardSelector: ".discard-answer"
,immediatelyShowMarkdownHelp:true
});


}
});














draft saved

draft discarded


















StackExchange.ready(
function () {
StackExchange.openid.initPostLogin('.new-post-login', 'https%3a%2f%2fmath.stackexchange.com%2fquestions%2f3044135%2ffinding-a-particular-solution-to-a-linear-pde%23new-answer', 'question_page');
}
);

Post as a guest















Required, but never shown

























0






active

oldest

votes








0






active

oldest

votes









active

oldest

votes






active

oldest

votes
















draft saved

draft discarded




















































Thanks for contributing an answer to Mathematics Stack Exchange!


  • Please be sure to answer the question. Provide details and share your research!

But avoid



  • Asking for help, clarification, or responding to other answers.

  • Making statements based on opinion; back them up with references or personal experience.


Use MathJax to format equations. MathJax reference.


To learn more, see our tips on writing great answers.




draft saved


draft discarded














StackExchange.ready(
function () {
StackExchange.openid.initPostLogin('.new-post-login', 'https%3a%2f%2fmath.stackexchange.com%2fquestions%2f3044135%2ffinding-a-particular-solution-to-a-linear-pde%23new-answer', 'question_page');
}
);

Post as a guest















Required, but never shown





















































Required, but never shown














Required, but never shown












Required, but never shown







Required, but never shown

































Required, but never shown














Required, but never shown












Required, but never shown







Required, but never shown







Popular posts from this blog

Plaza Victoria

In PowerPoint, is there a keyboard shortcut for bulleted / numbered list?

How to put 3 figures in Latex with 2 figures side by side and 1 below these side by side images but in...